Đến nội dung

Hình ảnh

Topic ôn luyện VMO 2015

vmo2015

  • Please log in to reply
Chủ đề này có 137 trả lời

#81
pndpnd

pndpnd

    Trung sĩ

  • Thành viên
  • 164 Bài viết

BÀi này khá quen, từ giả thiết thì ta có thể thu chứng minh được:
$ f(x)=4x^3-3x$ luôn, như vậy ta có điều phải chứng minh!

 từ giả thiết thì ta có thể thu chứng minh được:
$ f(x)=4x^3-3x$ luôn 

Là sao hả bạn



#82
WhjteShadow

WhjteShadow

    Thượng úy

  • Phó Quản lý Toán Ứng dụ
  • 1323 Bài viết

 

Bài 43: Cho $(O;R)$ và $(O';R')$ với R khác R' cắt nhau tại 2 điểm phân biệt $A$ và $B$. Đường thẳng $d$ tiếp xúc với $(O)$ tại $P$, với $(O')$ tại $Q$. Gọi $M$ và $N$ là hình chiếu của $P$ và $Q$ tương ứng trên $OO'$. $AM$ giao với (O) tại $M'$ , $AN$ giao với $(O')$ tại $N'$ ( M' và N' khác A). Chứng minh $M',N',B$ thẳng hàng.

onthivmoez.png

Vẽ $PQ$ cắt $OO'$ tại $D$. $AB$ cắt $OO'$ và $PQ$ lần lượt tại $K$ và $R$. Do $AB$ là trục đẳng phương của $(O)$ và (O')$ nên $R$ có cùng phương tích với $(O)$ và $(O')$, suy ra $RP=RQ$. Mặt khác do $AB\perp OO'\Rightarrow $RK//PM//QN\Rightarrow RK$ là đường trung bình của hình thang $PQNM$, vậy $K$ là trung điểm $MN\Rightarrow $ tam giác $AMN$ có $AK$ vừa là đường cao vừa là trung trực $\Rightarrow AM=AN$.

Mà $AM=MB,AN=BN$ nên $AM=AN=BM=BN$ hay $AMBN$ là hình thoi $\Rightarrow AM//BN$.

Xét phép vị tự tâm $D$ biến $(O)$ thành $(O')$, biến $P$ thành $Q$, $M$ thành $N$ mà $M\in (O), B\in (O')$, $MM'//BN$ nên nó cũng biến $M'$ thành $B$, hay $D,M',B$ thẳng hàng. Tương tự $D,N',B$ thẳng hàng, ta có đpcm.


“There is no way home, home is the way.” - Thich Nhat Hanh

#83
hoangtubatu955

hoangtubatu955

    Sĩ quan

  • Thành viên
  • 429 Bài viết

Hình học: 

Bài 45. Cho tam giác $ABC$ có trực tâm $H$. Đường cao $BE,CF$. $CF$ cắt đường tròn ngoại tiếp tam giác $ABC$ tại $M$. Gọi $X$ là điểm thỏa mãn

$\widehat{XMA}=\widehat{XHB}=90^o$. Gọi $K$ là điểm đối xứng của $B$ qua $CH$. Chứng minh rằng: $CK, XA, EF$ đồng quy!



#84
BlackSelena

BlackSelena

    $\mathbb{Sayonara}$

  • Hiệp sỹ
  • 1549 Bài viết
Bài 44 : Cho tam giác $ABC$ nội tiếp $(O)$, $E,F$ lần lượt thuộc cạnh $AC,AB$. $M,N,P$ lần lượt là trung điểm $EF,BE,CF$. $Q$ là hình chiếu của $O$ lên $EF$. Chứng minh rằng $M,N,P,Q$ đồng viên.

http://diendantoanho...-2013/?p=344028

#7

(ôi năm 2012....)


Bài viết đã được chỉnh sửa nội dung bởi BlackSelena: 24-11-2014 - 23:19


#85
hoangtubatu955

hoangtubatu955

    Sĩ quan

  • Thành viên
  • 429 Bài viết

ScreenHunter_23%20Nov.%2024%2023.49.jpg?

Câu này ngoài cách của anh Đạt, thực chất nó đơn giản là bài toán sau:
Từ một điểm $X$ nằm ngoài đường tròn $(O')$ vẽ tiếp tuyến $XQ$. $N$ là chân đường vuông góc từ $Q$ xuống $XO'$. A là một điểm bất kì nằm trên $(O')$. $AN$ cắt $(O')$ tại điểm thứ 2 là $N'$. Khi đó với $B$ là điểm đối xứng của $A$ qua $XO'$ thì $X,N',B$ thẳng hàng.
Chứng minh:
Gọi giao điểm của $XN'$ với $(O')$ là $B'$ ta chỉ cần chứng minh $AB'$ vuông góc $XO'$.
Chú ý rằng tứ giác: $ONN'B'$ nội tiếp, nên $\widehat{B'O'y}=\widehat{B'N'N}=\frac{1}{2}.\widehat{AO'B}$ vậy ta có điều phải chứng.


Bài viết đã được chỉnh sửa nội dung bởi hoangtubatu955: 25-11-2014 - 21:14


#86
LNH

LNH

    Bất Thế Tà Vương

  • Hiệp sỹ
  • 581 Bài viết

Bài 46: Cho tập $S$ gồm $1953$ điểm thỏa mãn $2$ điểm bất kì của $S$ cách nhau ít nhất $1$ cm. Chứng minh rằng: tồn tại tập con của $S$ có $217$ điểm sao cho $2$ điểm bất kì trong tập con đều có khoảng cách ít nhất $\sqrt{3}$ cm.



#87
binvippro

binvippro

    Trung sĩ

  • Thành viên
  • 193 Bài viết

Bài 47: Cho tứ giác $ABCD$ hai đường chéo $AC$ và $BD$ giao nhau tại $E.M,N$ thuộc $AB$ sao cho $AM=MN=NB$; $P,Q$ thuộc $CD$ sao cho $DP=PQ=QC$. $MQ$ giao $AC$ tại $K$. $NP$ giao $BD$ tại $L$. $MQ$ giao $NP$ tại $I$. Chứng minh rằng $EI$ luôn đi qua trung điểm của $KL$


Bài viết đã được chỉnh sửa nội dung bởi namcpnh: 09-12-2014 - 23:44


#88
deathavailable

deathavailable

    Thượng sĩ

  • Thành viên
  • 265 Bài viết

Bài 48: Tìm tất cả các hàm số $f: \mathbb{N}^*\rightarrow \mathbb{N}^*$ thỏa mãn  $f(f(n))=n^2$


Bài viết đã được chỉnh sửa nội dung bởi namcpnh: 09-12-2014 - 23:46

Ế là xu thế mang tầm cỡ quốc tế của các cấp bậc vai vế

 


#89
Hoang Tung 126

Hoang Tung 126

    Thiếu tá

  • Thành viên
  • 2061 Bài viết

Bài 49 : Cho dãy số $u_{n}$ thỏa mãn 

 

      $u_{1}=1,u_{n+1}=\frac{16u_{n}^3+27u_{n}}{48u_{n}^2+9}$

 

Đặt $S_{n}=\sum_{n=1}^{2015}\frac{1}{4u_{n}+3},n\in N,n\geq 1$.

 

     Tìm phần nguyên của $S_{n}$


Bài viết đã được chỉnh sửa nội dung bởi namcpnh: 09-12-2014 - 23:46


#90
binvippro

binvippro

    Trung sĩ

  • Thành viên
  • 193 Bài viết

Bài 50: Giải hệ phương trình $\left\{\begin{matrix} x\sqrt{y}+y\sqrt{x}+2(x+y-xy)=4 & \\ x\sqrt{x^2+3xy}+y\sqrt{y^2+3xy}=4 & \end{matrix}\right.$


Bài viết đã được chỉnh sửa nội dung bởi binvippro: 01-12-2014 - 12:07


#91
ducvipdh12

ducvipdh12

    Sĩ quan

  • Thành viên
  • 454 Bài viết

Bài 33 em giải như sau:

Ta xét 3 trường hợp sau:

TH 1:$\left | k \right |> 2$

Dùng BĐT sau:

$\left | u_{n+1} \right |\geq \left | k \right |.\left | u_{n} \right |-\left | u_{n-1} \right |> 2\left | u_{n} \right |-\left | u_{n-1} \right |\geqslant \left | u_{n} \right |$ nếu$\left | u_{n} \right |\geq \left | u_{n-1} \right |> 0$

Ta suy ra $\left | u_{0} \right |=\left | u_{1} \right |< \left |u _{2} \right |< ...$

TH 2: $\left | k \right |\leq 2$,k là số hữu tỉ nhưng k không là số nguyên,tức là $k=\frac{p}{q};p,q \epsilon \mathbb{Z}, q\geq 2,(p,q)=1$

Ta có $u_{2}=\frac{p}{q}u_{1}-u_{0}=\frac{-p-q}{q}$

Quy nạp theo n,giả sử $u_{i}=\frac{p_{i}}{q_{i-1}},p_{i}\in \mathbb{Z},(p_{i},q)=1;\forall i=1,2,...,n$ (1)

Khi đó $u_{n+1}=\frac{p}{q}u_{n}-u_{n-1}=\frac{p_{n+1}}{q^n}$ trong đó $p_{n+1}=p.p_{n}-q^2.p_{n-1} \in \mathbb{Z}$ thỏa mãn $(p_{n+1},q)=1$

Do $q\geqslant 2$, từ (1) suy ra $u_{n}\neq u_{m}$ với mọi $n\neq m$

Nói riêng $(u_{n})$ không là dãy số tuần hoàn.

TH 3: $\left | k \right |\leq 2,k\in \mathbb{Z}$ thì ta xét 5 giá trị của k thì dễ dàng tìm được có 4 giá trị của k thỏa mãn là -2,-1,0,1 

Cách của em không hay bằng cách của bạn Đạt nhưng đây cũng là 1 hướng tiếp cận bài toán dạng này  :luoi:


Bài viết đã được chỉnh sửa nội dung bởi ducvipdh12: 28-11-2014 - 22:47

FAN THẦY THÔNG,ANH CẨN,THẦY VINH :icon6: :icon6:

#92
hoangtubatu955

hoangtubatu955

    Sĩ quan

  • Thành viên
  • 429 Bài viết

Bài 50: Giải hệ phương trình $\left\{\begin{matrix} x\sqrt{y}+y\sqrt{x}+2(x+y-xy)=4 & \\ x\sqrt{x^2+3xy}+y\sqrt{y^2+3xy}=4 & \end{matrix}\right.$

Đầu tiên, ta có điều kiện là: $x,y \ge 0 $
+Nếu $x.y=0$ thì ta có $(x,y)=(2,0)$ và $(x,y)=(0,2)$ là nghiệm.
+Nếu $xy>0$ từ phương trình 2 bằng Cauchy ta thu được: $xy \le 1$
Từ đây, áp dụng bất đẳng thức Cauchy ta có:
 $\frac{1}{\sqrt{x}}+\frac{1}{\sqrt{y}} \ge 2 $ 

$\to x.\sqrt{x}+y.\sqrt{y}-2xy \ge 0 $, kết hợp phương trình (1) ta thu được: $x+y \le 2$.
Mặt khác, theo bất đẳng thức Cauchy-Schwarz ta có:
$4=x\sqrt{x^2+3xy}+y\sqrt{y^2+3xy} \le \sqrt{(x+y)(x^3+3x^2y+y^3+3xy^2)}=(x+y)^2$
Do đó $x+y \ge 2$
Như vậy $x+y=2$ và đẳng thức xảy ra khi và chỉ khi $x=y=1$, thử lại thấy $(x.y)=(1,1)$ là nghiệm của hệ.
Vậy hệ có các nghiệm là $(x,y)=(0,2); (x,y)=(2,0); (x,y)=(1,1)$



#93
ducvipdh12

ducvipdh12

    Sĩ quan

  • Thành viên
  • 454 Bài viết

Bài 51: Hãy tìm tất cả các tập A gồm hữu hạn số thực có tính chất sau:

Nếu x thuộc A thì $f(x)=x^{3}-3\left | x \right |+4$ cũng thuộc A

Bài 52: Có tồn tại hay không một đa giác đều 600 cạnh trong mặt phẳng tọa độ Descartes Oxy mà tọa độ các đỉnh của nó đều là số hữu tỉ?


Bài viết đã được chỉnh sửa nội dung bởi ducvipdh12: 28-11-2014 - 23:03

FAN THẦY THÔNG,ANH CẨN,THẦY VINH :icon6: :icon6:

#94
WhjteShadow

WhjteShadow

    Thượng úy

  • Phó Quản lý Toán Ứng dụ
  • 1323 Bài viết

Mình cũng đóng góp một bài mà mình đánh giá là khá hay :") 

Bài 48: Tìm tất cả các hàm số $f: \mathbb{N}^*\rightarrow \mathbb{N}^*$ thỏa mãn  $f(f(n))=n^2$

=================================

@LNH: nhớ ghi số bài nhé bạn :)

Đầu tiên dễ thấy $f$ đơn ánh.

Ta sẽ chứng minh 2 điều sau :

1. Chỉ tồn tại duy nhất $n=1$ để $f(n)=n$

2. Với mỗi số $n$ không chính phương thì có đúng 1 trong 2 mệnh đề sau đúng 

$\bullet$ $f(n)$ cũng không phải số chính phương.

$\bullet$ Tồn tại số $m$ không chính phương để $f(m)=n$.

Thật vậy, với $f(n)=n$ thì $f(f(n))=n=n^2\Rightarrow n=1$. Ta tập trung chứng minh điều 2 :

$\star$ Nếu mệnh đề thứ nhất đúng : 

Giả sử phản chứng, mệnh đề thứ 2 cũng đúng, lúc đó tồn tại $m$ không chính phương $f(m)=n\Rightarrow f(f(m))=f(n)=m^2$ (mâu thuẫn)

$\star$ Nếu mệnh đề thứ nhất sai : Lúc đó tồn tại $m\in \mathbb{Z}^{+}$ để $f(n)=m^2\Rightarrow f(n)=f(f(m))$ nên $n=f(m)$ (Do $f$ đơn ánh). Nếu $m$ là chính phương, $m=k^2$ thì $n=f(m)=f(k^2)=f(f(f(k)))=(f(k))^2$ (Mâu thuẫn do $n$ không chính phương).

Vậy tóm lại cả 2 điều trên đã được chứng minh. Từ đó ta suy ra tập các số nguyên dương không chính phương được chia thành các cặp $(m;n)$ ($m \neq n$ ) mà 

$$f \, : \, m\to n \to m^2\to n^2\geq m^4\to n^4\to .....$$

Và 2 cặp khác nhau $(m;n)$ , $(p;q)$ phải gồm 4 số khác nhau (Phản chứng nếu tồn tại 1 số giống nhau thì suy ra 2 cặp trùng nhau ngay !), và $f(n)$ với $n$ là số chính phương luôn được xác định từ $f(\sqrt{n})$. Như vậy hàm $f$ hoàn toàn xác định (vô số hàm $f$).

Bài toán tổng quát : Với hàm $g \, : \, \mathbb{N}^{*}\to \mathbb{N}^{*}$ nào thì tồn tại hàm $f \, : \, \mathbb{N}^{*}\to \mathbb{N}^{*}$ thỏa mãn :

$$f(f(n))=g(n) \,\,\, \forall n\in \mathbb{N}^{*}$$

Câu trả lời là hàm $g$ cần thỏa mãn 2 điều kiện sau : 

1, $g$ là đơn ánh.

2. Đặt $A=\{ g(n) \, | \, n \in \mathbb{N}^{*}\}$, $B=\mathbb{N}^{*}$ \ $A$, $C=\{n \, | \, g(n)=n\}$, thì $\forall \, m\in \mathbb{N}^{*}$, tồn tại $u\in \mathbb{N}^{*}$ và $n\in B$ thỏa mãn $m=g^{(u)}(n)$ (Hàm hợp).


Bài viết đã được chỉnh sửa nội dung bởi WhjteShadow: 29-11-2014 - 22:37

“There is no way home, home is the way.” - Thich Nhat Hanh

#95
pndpnd

pndpnd

    Trung sĩ

  • Thành viên
  • 164 Bài viết

Bài 53: Tìm số tự nhiên n nhỏ nhất, $n>0$ sao cho tổng $\sum_{i=1}^{n}p_{i}^{2000} $ chia hết cho $120$ với $p_{1};p_{2};p_{3};....$ là các số nguyên tố tùy ý lớn hơn 10


Bài viết đã được chỉnh sửa nội dung bởi pndpnd: 29-11-2014 - 18:07


#96
pndpnd

pndpnd

    Trung sĩ

  • Thành viên
  • 164 Bài viết

Bài 54:  Cho $a,b,c>0$. Giải hệ:

$\left\{\begin{matrix}\frac{a}{x}-\frac{b}{z}=c-xz & & \\ \frac{b}{y}-\frac{c}{x}=a-xy & & \\ \frac{c}{z}-\frac{a}{y}=b-yz & & \end{matrix}\right.$



#97
chardhdmovies

chardhdmovies

    Thiếu úy

  • Thành viên
  • 638 Bài viết

Bài 54:  Cho $a,b,c>0$. Giải hệ:

$\left\{\begin{matrix}\frac{a}{x}-\frac{b}{z}=c-xz & & \\ \frac{b}{y}-\frac{c}{x}=a-xy & & \\ \frac{c}{z}-\frac{a}{y}=b-yz & & \end{matrix}\right.$

qui đồng rồi cộng các phương trình ta được $axy+byz+czx=x^2y^2+y^2z^2+z^2x^2$        $(1)$

$c.PT(1)+a.PT(2)+b.PT(3)\Rightarrow axy+byz+czx=a^2+b^2+c^2$                             $(2)$

cộng $(1)$ và $(2)$ ta được $(xy-a)^2+(yz-b)^2+(zx-c)^2=0\Rightarrow \left\{\begin{matrix} xy=a\\yz=b \\zx=c \end{matrix}\right.$

phần còn lại ok rồi

Spoiler

 

NTP


                                                                                    chúng tôi là 3 người từ lớp 10 cá tính:NRC,NTP,A-Q


#98
BlackSelena

BlackSelena

    $\mathbb{Sayonara}$

  • Hiệp sỹ
  • 1549 Bài viết

Bài 55:

Tìm hàm $f: \mathbb{R}^+ \rightarrow \mathbb{R}^+$ thoả:

$f(xy) = f(x+y)(f(x)+f(y)) \ \forall x,y > 0$



#99
LNH

LNH

    Bất Thế Tà Vương

  • Hiệp sỹ
  • 581 Bài viết

Bài 40 : Tìm hàm $f$ liên tục $\mathbb{R}\to \mathbb{R}$ sao cho $f(x)-f(y)\in \mathbb{Q}\forall x-y \in \mathbb{Q}$

Đặt $g(x)=f(x)-f(0)$, ta có $g(0)=0$ và $g(x)-g(y) \in \mathbb{Q}\Leftrightarrow x-y \in \mathbb{Q}$

Cố định số hữu tỉ $r$, xét $h(x)=g(x+r)-g(x)$

Ta có $h(x) \in \mathbb{Q}, \forall x \in\mathbb{R}$

Mà $h$ liên tục trên $\mathbb{R}$ nên $h(x)=const$

$h\left ( 0 \right )=g\left ( r \right )\Rightarrow g\left ( x+r \right )=g\left ( x \right )+g\left ( r \right ), \forall r \in \mathbb{Q}$

Từ đây, ta chứng minh được $g\left ( x \right )=xg(1), \forall x \in \mathbb{Q}$

Mà $g$ liên tục trên $\mathbb{R}$ nên $g\left ( x \right )=xg\left ( 1 \right ),\forall x \in \mathbb{R}$

Suy ra $f\left ( x \right )\equiv ax+b$ với $a,b \in \mathbb{Q}$ (thử lại thấy thỏa mãn)



#100
Bui Ba Anh

Bui Ba Anh

    Thiếu úy

  • Thành viên
  • 562 Bài viết

Bài 56: Các học sinh được phát bài kiểm tra, mỗi môn một bài trong $n$ ($n \geq 3$) môn học. Biết rằng với mỗi môn bất kỳ có $3$ học sinh đạt điểm tối ưu, còn $2$ môn tùy ý thì có $1$ học sinh đạt điểm tối ưu cho cả $2$ môn. Xác định $n$ bé nhất sao cho từ điều kiện suy ra có đúng $1$ học sinh đạt điểm tối ưu cho cả $n$ môn


NgọaLong





Được gắn nhãn với một hoặc nhiều trong số những từ khóa sau: vmo2015

1 người đang xem chủ đề

0 thành viên, 1 khách, 0 thành viên ẩn danh